El Principio de Exclusión de Pauli para más de dos partículas, aplicado al Potencial de Coulomb

Descargo de responsabilidad : hice una pregunta muy similar antes (que puedo proporcionar si lo desea, pero no debe contener nada que no se indique aquí), pero fue rechazado y finalmente eliminado por razones desconocidas para mí (por lo tanto, si voy a rechazar esta pregunta, ¡dime por qué!). Sin embargo, aunque no obtuve una respuesta a mi pregunta, los comentarios ayudaron a aclarar un poco el problema, por lo que ahora estoy tratando de formular mi pregunta con un poco más de claridad. Además, lamento que esta haya resultado ser una pregunta tan larga, pero sentí la necesidad de ser lo más precisa posible sobre mi pregunta, para que no vuelva a suceder lo mismo.

Cuando se resuelve la ecuación de Schrödinger para el potencial de Coulomb, generalmente se comienza dividiendo la función de onda en partes temporales, angulares y radiales separadas, lo que se justifica con la afirmación de que todas las soluciones posibles de la ecuación de Schrödinger se pueden obtener como combinaciones lineales de las funciones especiales unos. Luego, la parte angular se divide aún más en dos partes más, y se obtienen conjuntos discretos de soluciones para ambas, de modo que, al final, las soluciones (dividibles) de la ecuación angular son exactamente los armónicos esféricos, que pueden ser parametrizados de forma única por dos números enteros yo y metro .

Lo que me había molestado de esto antes era que, al dividir la ecuación angular en dos partes, uno tiene que elegir un eje a lo largo del cual construir el sistema de coordenadas polares, lo que luego da como resultado que los armónicos esféricos estén orientados alrededor de ese eje, de modo que en Al final, la elección de este eje influye en el conjunto de soluciones obtenido. En realidad, el conjunto de soluciones de la ecuación angular debe cerrarse bajo rotación, ya que la ecuación en sí también es rotacionalmente simétrica.

Los comentarios a mi pregunta anterior han confirmado mis conjeturas de que estas variantes rotadas de la ecuación de Schrödinger aparecen exactamente como esas combinaciones lineales de las soluciones fundamentales obtenidas al dividir la función angular, de modo que, al final, las soluciones a la ecuación angular están de hecho cerrados por rotación. Sin embargo, esto significa que el conjunto de soluciones no es de hecho discreto y no puede ser descrito solo por los dos números enteros yo y metro .

Esto, entonces, es exactamente la parte que me confunde, ya que en todas partes de la literatura se afirma que el estado de un electrón dentro de un átomo (si aproximamos el potencial como un potencial de Coulomb e ignoramos las interacciones entre electrones, y asumimos que el estado es ser uno de energía definida (o, de manera equivalente, independiente del tiempo) se puede describir de manera única dando los cuatro números enteros norte (que da el nivel de energía o, de manera equivalente, especifica una solución de la ecuación radial), yo , metro (que especifican la parte angular), y s (El giro). Sin embargo, si lo que me dijeron es correcto, incluso si asumimos que nuestro electrón ocupa una capa específica y, por lo tanto, lo factorizamos en partes radiales y angulares, el estado de un electrón general aún no puede describirse mediante estos cuatro números, ya que estos aparentemente ignoran las posibles combinaciones lineales, o equivalentemente, versiones rotadas del conjunto discreto de armónicos esféricos.

Por otra parte, esta afirmación de que el estado de un electrón se describe únicamente por norte , yo , metro , y s luego se usa para aplicar el principio de exclusión de Pauli para derivar el hecho de que ciertas capas de electrones de un átomo solo pueden estar ocupadas por una cierta cantidad de electrones, ya que solo hay muchos estados distintos posibles en los que puede estar el electrón.

Esto no concuerda con mi impresión previa de que debería haber infinitos estados posibles para un electrón en una capa dada, debido a la simetría rotacional continua. Por ejemplo, según tengo entendido, se podría tomar un estado posible de un electrón en la segunda capa (cuya parte angular no debería ser constante) y rotar su parte angular en ángulos pequeños, obteniendo cada vez una función de onda diferente. De acuerdo con la forma en que siempre he escuchado que el Principio de exclusión de Pauli se establece antes (dos fermiones indistinguibles no pueden tener EXACTAMENTE la misma función de onda y espín (es decir, el mismo estado)), no debería prohibir esta configuración.

Esto me parece una contradicción, así que me gustaría saber dónde está el error en mi razonamiento. Mi conjetura personal es que mi idea del Principio de Inclusión de Pauli para más de dos partículas es incorrecta, y que en realidad dice algo similar al conjunto de estados de fermiones indistinguibles que siempre son linealmente independientes o mutuamente ortogonales o algo similar. Eso sería algo intuitivo (y la versión 'linealmente independiente' se reduce a la declaración clásica para dos partículas) y creo que debería resolver este problema (ya que, si no recuerdo mal, las soluciones específicas para los armónicos esféricos forman una base ortogonal), pero no he podido encontrar tal formulación del principio en ninguna parte (de hecho, siempre ha sido la misma afirmación que he dado arriba) y no he llegado muy lejos tratando de demostrarlo, ya que todavía no estoy seguro de cómo se representaría algebraicamente el espacio de Hilbert correspondiente a un sistema conjunto de fermiones indistinguibles. (De hecho, aún no sé cómo formular rigurosamente la indistinguibilidad dados los espacios de Hilbert, por lo que si alguien pudiera ayudarme con eso, también estaría muy agradecido).

Un intento de la formulación final de mi pregunta: si el conjunto de posibles estados de electrones en una capa dada de un átomo es realmente infinito, ¿cómo se usa el principio de exclusión de Pauli (y cuál es su declaración precisa para más de dos partículas? ) para deducir que solo un número limitado de electrones puede estar en esta capa a la vez?

Respuestas (1)

"Realmente, el conjunto de soluciones de la ecuación angular debe cerrarse bajo rotación, ya que la ecuación en sí también es rotacionalmente simétrica". esto no es correcto Las soluciones deben transformarse como una representación del grupo de rotación. Deben ser covariantes, no necesariamente invariantes. Solo un caparazón completo es invariante.

"debería haber infinitos estados posibles para un electrón en una capa dada, debido a la simetría rotacional continua" de hecho hay infinitos, pero solo 2 ( 2 yo + 1 ) son linealmente independientes. Usted es libre de elegir cuáles eligiendo el llamado eje de cuantificación.

Una vez seleccionado el conjunto de orbitales linealmente independientes, se pueden llenar siguiendo el principio de exclusión.

Tenga en cuenta que un cálculo preciso de los orbitales requiere la llamada teoría de campo autoconsistente.

En cuanto a su primer punto, esa podría haber sido una redacción inexacta de mi parte, no quise decir que todas las soluciones deberían ser rotacionalmente simétricas, solo que una versión rotada de una solución vuelve a ser una solución. Creo que eso es lo que estás diciendo allí también.
A su segundo punto: ¿Significa eso que mi interpretación del Principio de exclusión de Pauli (que una colección de fermiones indistinguibles no puede tener estados linealmente independientes) es (aproximadamente) correcta? Al menos eso es lo que creo que estás diciendo al elegir el eje de cuantificación.
Linealmente dependiente. Probablemente un error tipográfico.
¿Te ha ayudado mi respuesta a aclarar tus dudas? ¿Tiene alguna pregunta restante?
Creo que sí, ya que la contradicción ya está resuelta. Tampoco puedo pensar en más preguntas, así que gracias de nuevo :)
Espera, en realidad acabo de pensar en algo. Si el principio de exclusión de Pauli se cumple como dijiste, ¿significaría eso que el espacio vectorial de todos los estados posibles de k fermiones indistinguibles es el k -ésima potencia exterior del espacio de estados de uno solo? Nuevamente, es algo que intuitivamente parece que podría funcionar, pero no estoy completamente seguro de cómo probarlo o derivarlo correctamente.
Eso suena correcto para mí. Eso es un espacio muy grande sin embargo.